Азиатско-Тихоокеанская математическая олимпиада, 1991 год


$a_1$, $a_2$, $\dots$, $a_n$, $b_1$, $b_2$, $\dots$, $b_n$ — положительные вещественные числа такие, что $a_1 + a_2 + \dots + a_n = b_1 + b_2 + \dots + b_n$. Докажите, что $$ \frac{{a_1^2}}{{{a_1} + {b_1}}} + \frac{{a_2^2}}{{{a_2} + {b_2}}} + \dots + \frac{{a_n^2}}{{{a_n} + {b_n}}} \geq \frac{{{a_1} + {a_2} + \dots + {a_n}}}{2}. $$
посмотреть в олимпиаде

Комментарий/решение:

  1
2019-02-25 14:34:11.0 #

$$ \forall a_1,a_2,...,a_n \in \mathbb{R}^+, \quad \forall b_1,b_2,...,b_n \in \mathbb{R}^+:$$

$$ \sqrt{\Bigg( \sum_{k=1}^n \Bigg(\frac{a_k}{\sqrt{a_k+b_k}}\Bigg)^2 \Bigg) }\sqrt{\Bigg(\sum_{k=1}^n(\sqrt{a_k+b_k})^2\Bigg)} \geq \sum_{k=1}^n a_k \Rightarrow $$

$$ \Rightarrow \Bigg( \sum_{k=1}^n \Bigg(\frac{a_k}{\sqrt{a_k+b_k}}\Bigg)^2 \Bigg) \Bigg(\sum_{k=1}^n(\sqrt{a_k+b_k})^2\Bigg) \geq \Bigg( \sum_{k=1}^n a_k \Bigg)^2$$

$$ \sum_{k=1}^n(\sqrt{a_k+b_k})^2=\sum_{k=1}^n(a_k+b_k)=2\sum_{k=1}^n a_k \Rightarrow$$

$$\Rightarrow \Bigg( \sum_{k=1}^n \Bigg(\frac{a_k}{\sqrt{a_k+b_k}}\Bigg)^2 \Bigg) \Bigg(2\sum_{k=1}^n a_k\Bigg) \geq \Bigg( \sum_{k=1}^n a_k \Bigg)^2\Rightarrow$$

$$\Rightarrow \Bigg( \sum_{k=1}^n \Bigg(\frac{a_k}{\sqrt{a_k+b_k}}\Bigg)^2 \Bigg) \geq \frac{1}{2} \cdot \Bigg( \sum_{k=1}^n a_k \Bigg)$$

Причём равенство достигается тогда и только тогда, когда векторы $\Bigg( \frac{a_1}{\sqrt{a_1+b_1}}, \frac{a_2}{\sqrt{a_2+b_2}},...,\frac{a_n}{\sqrt{a_n+b_n}}\Bigg)^T$ и $ (\sqrt{a_1+b_1}, \sqrt{a_2+b_2},...,\sqrt{a_n+b_n})^T$ пропорциональны:

$$\forall k=1,...,n: \qquad \frac{\frac{a_k}{\sqrt{a_k+b_k}}}{\sqrt{a_k+b_k}}=\lambda \Rightarrow a_k \Rightarrow \frac{a_k}{a_k+b_k}=\lambda \Rightarrow b_k=\Bigg(\frac{1}{\lambda}-1 \Bigg) a_k \Rightarrow \sum_{k=1}^nb_k=\Bigg(\frac{1}{\lambda}-1 \Bigg) \sum_{k=1}^n a_k =\sum_{k=1}^n a_k \Rightarrow$$ $$ \Rightarrow\frac{1}{\lambda}-1=1 \Rightarrow \lambda =\frac{1}{2} \Rightarrow a_k=b_k$$

  2
2019-02-25 15:24:01.0 #

По неравенству Коши-Буняковского

$\dfrac{a_{1}^2}{a_{1}+b_{1}} + \dfrac{a_{2}^2}{a_{2}+b_{2}}+...+\dfrac{a_{n}^2}{a_{n}+b_{n}} \geq \dfrac{(a_{1}+a_{2}+...+a_{n})^2}{a_{1}+a_{2}+...+a_{n}+b_{1}+b_{2}+...+b_{n}} = \dfrac{(a_{1}+a_{2}+...+a_{n})^2}{2(a_{1}+a_{2}+...+a_{n})} = \dfrac{a_{1}+a_{2}+...+a_{n}}{2}$

  1
2021-05-22 23:28:44.0 #

какая-то примитивная задача для АТМО

  2
2021-05-23 00:56:19.0 #

Так 91 год же, тогда задачи намного легче были

  1
2021-06-13 13:33:25.0 #

тогда КБШ не было, или он был в узких кругах, вот и прикол задачи был наверное ;)

  0
2021-06-13 20:42:06.0 #

На самом деле задача решается лишь одной подстановкой КБШ дробного вида, но стоит отметить что в те года он был популярен лишь в узких кругах, или его и вовсе не придумали к тому времени.Покажу решение через $AM $$\geq$ $GM$.

Для каждого вида возьмём и добавим $\frac{a_{1}+b_{1}}{4}$ и используем сам $AM$ $\geq$ $ GM$.Дальше уже просто подсчёт ;)